Вы находитесь на странице: 1из 2

KEITH DEVLIN: Introduction to Mathematical Thinking (Fall 2013)

Problem Set Week 8

This problem set focuses on material covered in Week 8 (Lecture 10), so I recommend you to watch the lecture and attempt Assignment 10 (both parts) before submitting your answers. The deadline for completing (and submitting) the problem set is Monday October 28 at 9:00AM US-PDT. Note that you can save your entries as you work through the problems, and can change them at any time prior to submission, but once you submit your answers no further changes are possible.

1. Say which of the following are true. (Leave the box empty to indicate that its false.) A set A of reals can have at most one least upper bound. If a set A of reals has a lower bound, it has innitely many lower bounds. If a set A of reals has both a lower bound and an upper bound, then it is nite. 0 is the least upper bound of the set of negative integers, considered as a subset of the reals.

2. Which of the following say that b is the greatest lower bound of a set A of reals? (Leave the box empty to indicate that it does not say that.) b a for all a A and if c a for all a A, then b c. b a for all a A and if c a for all a A, then b > c. b < a for all a A and if c < a for all a A, then b c. b < a for all a A and if c a for all a A, then b c. b a for all a A and if > 0 there is an a A such that a < b + .

3. The Sandwich Theorem (also sometimes called the Squeeze Theorem ) says that if {an } n=1 , {bn }n=1 , {cn }n=1 are sequences such that, from some point n0 onwards,

an bn cn , and if
n

lim an = L , lim cn = L,
n

then

{bn } n=1

is convergent and
n

lim bn = L.

Taking the Sandwich Theorem to be correct (which it is), grade the following proof using the course rubric. sin2 n Theorem lim =0 n 3n Proof: For any n, sin2 n 1 0 n 3n 3 Clearly, lim
n

1 = 0. Hence, by the Sandwich Theorem, 3n sin2 n =0 n 3n lim

as required. 1

4. Is the following proof of the Sandwich Theorem correct? Grade it according to the course rubric.
Theorem (Sandwich Theorem) Suppose {an } n=1 , {bn }n=1 , {cn }n=1 are sequences such that, from some point n0 onwards, an bn cn .

Suppose further that


n

lim an = L , lim cn = L.
n

Then

{bn } n=1

is convergent and
n

lim bn = L.

Proof: Since lim an = L, we can nd an integer n1 such that


n

n n1 |an L| < Since lim cn = L, we can nd an integer n2 such that


n

n n2 |cn L| < Let M = max{n0 , n1 , n2 }. Then nM ( < an L < ) ( < cn L < ) < an L bn L cn L < < bn L < |bn L| <
n

(using an bn cn )

By the denition of a limit, this proves that {bn } n=1 is convergent and lim bn = L, as required. 5. Evaluate this purported proof, and grade it according to the course rubric. n+1 1 Theorem lim = . n 2n + 1 2 1 Proof: Let > 0 be given. Choose N large enough so that N . 2 Then, for n N , n+1 1 2n + 1 2 = 2(n + 1) (2n + 1) 2(2n + 1) 1 2(2n + 1) 1 2(2n + 1) 1 2n + 1 1 1 2n 2N

= < <

By the denition of a limit, this proves the theorem.

Вам также может понравиться